Search found 1179 matches


Let the number of bags be n and the number of marbles in each bag except one be x. So one bag has x+1 marble. Or (n-1)*x + (x+1) = 2001. Or nx + 1 = 2001. Or nx = 2000. Note that both n and x are positive integers and are factors of 2000. First consider (1) alone . The factors of 2000 between 13 and...

by Rahul@gurome

Sat Feb 26, 2011 3:14 am
Forum: Data Sufficiency
Topic: Box, Bags, and Marbles!!
Replies: 3
Views: 1371

Rahul, Just two follow up questions 1) We know 30/60/90 has lengths x: xroot(3): 2x. If we know the length of a triangle is x: xroot(3) : 2x do we know if its a 30/60/90? 2) In the previous problem we know two sides of a triangle and one angle. What additional info.will we need to deem that the tri...

by Rahul@gurome

Fri Feb 25, 2011 10:54 pm
Forum: Data Sufficiency
Topic: JH- Geo 2
Replies: 3
Views: 1397

Solution: Obviously, each statement alone is not sufficient to answer the question. We next combine both the statements together and check. On combining we know that AB is 5 and AC is 5*sqrt2. Now, in a right angled triangle, hypotenuse is the largest size. Now since AC > AB, AB can never be the hyp...

by Rahul@gurome

Fri Feb 25, 2011 10:23 pm
Forum: Data Sufficiency
Topic: JH- Geo 2
Replies: 3
Views: 1397

Solution: The container has 40 liters of milk. From this, 4 liters of milk was taken away and replaced with water. So, now we have 36 liters of milk and 4 liters of water. Hence the new mixture has milk and water in the ratio 36:4 or 9:1. So, now if we withdraw 4 liters, the remaining 36 liters has ...

by Rahul@gurome

Fri Feb 25, 2011 9:44 pm
Forum: Problem Solving
Topic: Alligation?
Replies: 5
Views: 2528

In a sequence of terms in which each term is three times the previous term, what is the fourth term? (1) The first term is 3. (2) The second-to-last term is 3^10. A Does somebody know what is the meaning of the second statement?, what can I get from it.? Thank Solution: Consider first (1) alone. Th...

by Rahul@gurome

Wed Feb 02, 2011 7:02 pm
Forum: Data Sufficiency
Topic: The second-to-last term
Replies: 2
Views: 1217

Usually I'm ok with these but this one threw me for a loop. Could anyone explain the mistake in my approach? Take the unrestricted combinations and subtract out the restricted combinations Unrestricted = 7! = 5040 possible combinations Restriction 1: She won't sit in the first seat. How many ways c...

by Rahul@gurome

Wed Feb 02, 2011 6:52 pm
Forum: Problem Solving
Topic: Probability--suzan
Replies: 7
Views: 2782

I'm confused here... The range isn't inclusive? I counted 1 through 7, inclusive. But to find the range you simply take max - min? Thanks! You don't have to take the number of elements between 1 and 7 inclusive. You have to take the difference between maximum positive value which is 7 and minimum p...

by Rahul@gurome

Wed Feb 02, 2011 6:44 pm
Forum: Problem Solving
Topic: Range of integers ques
Replies: 7
Views: 1797

quaktar0004 wrote:Why isn't "0" included in this range? I too thought the answer should be 7: 7-0=7. Please advise.
The reason is that zero is considered neither positive or negative.
You have to take only the positive integers while calculating the range.

by Rahul@gurome

Tue Feb 01, 2011 6:53 pm
Forum: Problem Solving
Topic: Range of integers ques
Replies: 7
Views: 1797

Note that we have been asked the range for positive integers in the list. Range is the difference between maximum and minimum value. The integers in the list are -4, -3, -2, -1, 0, 1, 2, 3, 4, 5, 6, 7. The minimum positive value is 1 and the maximum positive value is 7. So the range of positive inte...

by Rahul@gurome

Tue Feb 01, 2011 12:43 am
Forum: Problem Solving
Topic: Range of integers ques
Replies: 7
Views: 1797

Solution: n * 2^5 * 6^2 * 7^3 = n * 2^5 * 2^2 * 3^2 * 7^3 = n * 2^7 * 3^2 * 7^3. Since two 3's are there in the above expression, we need one 3 more to get 3^3 as factor. Also since 5 is not there in the expression, we need two 5's as factors. So, the minimum value of n is 5^2 * 3 = 75. The correct ...

by Rahul@gurome

Sun Jan 30, 2011 6:36 pm
Forum: Problem Solving
Topic: prime factorization
Replies: 1
Views: 1107

Consider first (1) alone. It means lx-3l >= 0. This will be true for infinite many values of x. So, statement (1) alone is not sufficient to answer the question. We next consider statement (2) alone. This means lx-3l <= 0. Since, modulus cannot be negative, lx-3l=0. Or x = 3. So, (2) alone is suffic...

by Rahul@gurome

Tue Jan 25, 2011 9:00 pm
Forum: Data Sufficiency
Topic: If y ≥ 0, What is the value of x?
Replies: 2
Views: 1288

@kvcpk : Thanx for pointing it out. Edited the reply.

by Rahul@gurome

Mon Jan 24, 2011 10:29 pm
Forum: Data Sufficiency
Topic: Five DS questions for practice today. Try !
Replies: 8
Views: 1561

kvcpk wrote:Are you sure about this question?

if a,b,c are positive, I dont think 1/a + 1/b + 1/c can ever be 0.
You're correct.
Ideally the question should declare a, b, c as non-zero integers.

by Rahul@gurome

Mon Jan 24, 2011 10:07 pm
Forum: Data Sufficiency
Topic: Five DS questions for practice today. Try !
Replies: 8
Views: 1561

... i don't have a very comfortable understanding of standard deviations. For a data set, standard deviation is the measure of dispersion or variation of the data from the mean or average. A low standard deviation indicates that the data points tend to be very close to the mean, whereas high standa...

by Rahul@gurome

Mon Jan 24, 2011 9:57 pm
Forum: Data Sufficiency
Topic: Question from MGMAT CAT Test
Replies: 3
Views: 2428

5. If n is a positive integer that is less than 10, what is the value of n? 1) n is the tenths digit in the decimal representation of 1/n. 2) n is the hundredths digit in the decimal representation of 1/n. I believe the original question is more like the above. Now, as n < 10, n can be any of the f...

by Rahul@gurome

Mon Jan 24, 2011 9:35 pm
Forum: Data Sufficiency
Topic: Five DS questions for practice today. Try !
Replies: 8
Views: 1561